Math Challenge - July 2019

In summary, we discussed various mathematical problems and solutions, including showing inequalities, finding roots of equations, calculating integrals, and proving properties of matrices and other mathematical objects. Some of the problems required the use of calculus and other advanced techniques, while others could be solved with basic concepts. We also discussed the importance of writing in LaTeX for better readability and understanding.
  • #1
fresh_42
Mentor
Insights Author
2023 Award
18,994
23,992
Questions

1.
(solved by @Flatlanderr , solved by @lriuui0x0 ) Show that ##\frac{\pi}{4} + \frac{3}{25} \lt \arctan \frac{4}{3} \lt \frac{\pi}{4} + \frac{1}{6}##

2. (solved by @nuuskur ) Show that the equation ##x + x^3 + x^5 + x^7 = {c_1}^2 (c_1 - x) + {c_2}^2 (c_2 - x)## where ##c_1, c_2 \in \mathbb{R}##, has only one real root.

3. (solved by @KnotTheorist ) Let ##A = \begin{bmatrix}
7 & 3\\
3 & -1
\end{bmatrix}##. Find an orthogonal matrix ##P## such that ##D = P^{-1} A P## is diagonal.

4. (solved by @nuuskur ) Using Calculus, show that the roots of the equation ##x^5 + ax^4 + bx^3 + cx^2 + dx + e = 0## cannot be all real if ##2a^2 \lt 5b##

5. Consider the projective space ##P^m(F_q) = P(F_q^{m+1})##. How many (t+1)-dimensional spaces in this projective space contain a given t-dimensional space?

6. (solved by @nuuskur ) Let ##C## be a subspace of ##F_q^n##. We call ##C## cyclic if ##C## contains its right shifts, i.e. ##x = (c_1, \dots, c_n) \in C \implies \vec{x} = (c_n, c_1 \dots, c_{n-1}) \in C##. Show that there is a correspondence between the cyclic subspaces of ##F_q^n## and the ideals of the quotient ring ##F_q[X]/(X^n-1)##.

7. (solved by @nuuskur ) Let ##(W_t)_{t\geq 0}## be a Brownian motion on a suitable probability space. Put ##X:= \int_{0}^t W_s^2 ds##. Calculate ##E(X).## (changed, 7/5/19)

8. (solved by @nuuskur ) Calculate the Galois group of the splitting field of ##x^4 - 2x^2 -2## over ##\mathbb{Q}##. Determine the isomorph type of the Galois group.

9. Prove that every symmetric and positive definite matrix ##A\in \mathbb{M}(n,\mathbb{R})## can be uniquely written as ##A=L\cdot L^\tau##, where ##L## is a lower triangular matrix with positive diagonal elements.
Calculate ##L## for ##A=\begin{bmatrix}4 &2& 4& 4\\
2& 10& 17& 11\\
4& 17& 33& 29\\
4& 11& 29& 39\end{bmatrix}##.

10. (solved by @nuuskur ) Let ##L\subseteq H## be a nonempty, closed, and convex set in a Hilbert space. Prove that there is an element of minimal norm in ##L##.

245983


11. (solved by @Physics lover ) Solve the following integral
$$I = \int_0^{\pi/2} \frac{\sin^{2019} x}{\cos^{2019} x + \sin^{2019} x}dx$$

12. (solved by @Pi-is-3 ) Is ##N:=21^{39}+39^{21}## divisible by ##45##? Why, why not?

13. (solved by @Pi-is-3 ) Let ##0<u,v,w<1##. Show that among the numbers ##u(1-v)\, , \,v(1-w)\, , \,w(1-u)## is at least one value not greater than ##\dfrac{1}{4}\,.##

14. (solved by @JosephFG ) What is the ratio between the red and the blue area where ##P## and ##Q## are one fifth away from the vertices?

245984


15. (solved by @Pi-is-3 ) In what ratio does the circumference of the circle divide the left and right sides of the square?

245986
 
Last edited:
  • Like
Likes Pi-is-3, YoungPhysicist, member 587159 and 1 other person
Physics news on Phys.org
  • #2
My Solution to 3
This is just a simple eigenvalue problem, where P is just the matrix of eigenvectors, which if the eigenvectors themselves are normalized, will become an orthogonal matrix. And then it will diagonalize to its eigenvalues.
We can get that the eigenvalues of A are 8 and -2, then solving for the eigenvectors, ensuring that they are normalized, we get v1 = 1/√10 (3 1) and v2 = 1/√10 (-1 3).
P =
\begin{pmatrix}
3/√10 & -1/√10 \\
1/√10 & 3/√10
\end{pmatrix}

And then P-1 = PT. Then solving for D, we get D = P-1 A P = PT A P =
\begin{pmatrix}
8 & 0 \\
0 & -2
\end{pmatrix}
 
  • Like
Likes QuantumQuest
  • #3
Question 1:

##\frac{\pi}{4} = \arctan 1##

Therefore we get ##\frac{3}{25} < \arctan \frac{4}{3} -\arctan 1 < \frac{1}{6}##.

Since

$$
\begin{aligned}
\tan (a - b) &= \frac{\tan a - \tan b}{1 + \tan a \tan b} \\
a - b &= \arctan \frac{\tan a - \tan b}{1 + \tan a \tan b}
\end{aligned}
$$

Let ##a = \arctan x## and ##b = \arctan y##,

we have

$$
\arctan x - \arctan y = \arctan \frac{a - b}{1 + ab}
$$

Therefore we need to prove ##\frac{3}{25} < \arctan \frac{1}{7} < \frac{1}{6}##.

Expand ##\arctan x## with power series, we have ##\arctan x = \sum (-1)^k \frac{x^{2k + 1}}{2k + 1}\; (k \ge 0)##. Denote partial sum as ##S_k##, since the power series is an alternating series whose absolute term value is monotonically decreasing, we can prove the greatest partial sum is ##S_0##, the lowest partial sum is ##S_1##. Therefore ##x - \frac{x^3}{3} < \arctan x < x##. Put in value ##\frac{1}{7}##, we get the inequality.

To prove ##S_0## and ##S_1## are maximum and minimum partial sum respectively, first note that ##S_{2k} > S_{2k + 1}##, then note ##S_{2k} > S_{2k + 2}## and ##S_{2k + 1} < S_{2k + 3}##.
 
  • #4
lriuui0x0 said:
Let ##a = \arctan x## and ##b = \arctan y##,

we have

## \arctan x - \arctan y = \arctan \frac{a - b}{1 + ab}##

How did you get this?
 
  • #5
Hi QuantumQuest,

So given ##a=\arctan x## and ##b=\arctan y##, put ##a## and ##b## into previous equation, we get

$$
\begin{aligned}
a − b &= \arctan \frac{\tan a − \tan b}{1+ \tan a \tan b} \\
\arctan x - \arctan y &= \arctan \frac{\tan \arctan x - \tan \arctan y}{1 + (\tan \arctan x)(\tan \arctan y)} \\
\arctan x - \arctan y &= \arctan \frac{x - y}{1 + xy}

\end{aligned}
$$
 
  • Like
Likes QuantumQuest
  • #6
Some ideas for the second problem.
Replace [itex]c_1 =: u, c_2 =: v[/itex]. Assume there is a (real) root [itex]r[/itex] for the polynomial. Transform initial problem to comparison of coefficients as follows.
[tex]
\begin{align*}
&x^7 + x^5 + x^3 + (1+u^2 + v^2)x - u^3 - v^3 \\ = &(x-r)(a_6x^6 + a_5x^5 + a_4x^4 + a_3x^3 + a_2x^2 + a_1x + a_0)
\end{align*}
[/tex]
It is now a matter of finding what [itex]r[/itex] is in terms of the parameters [itex]u,v[/itex]. If there is no branching, then [itex]r[/itex] is necessarely unique.

The coefficients are restricted as follows.
[tex]
\begin{align*}
&a_6 = 1 \\
&a_5 = a_6r
&a_4 = a_5r + 1 \\
&a_3 = a_4r
&a_2 = a_3r + 1 \\
&a_1 = a_2r
&a_0 - a_1r = 1+u^2 + v^2 \\
&a_0r = u^3 + v^3
\end{align*}
[/tex]
Using
[tex]
\begin{cases}a_0r = u^3 + v^3 \\ a_0 - a_1r = 1+ u^2+v^2 \end{cases}
[/tex]
multiplying both sides of the second equality by [itex]r[/itex] we get
[tex]
a_0r - a_1r^2 = (1+u^2+v^2)r \iff a_1r^2 + (1+u^2+v^2)r - (u^3+v^3) = 0.
[/tex]
For [itex]r[/itex] to be unique, we require the discriminant
[tex]
(1+u^2+v^2)^2 + 4a_1(u^3+v^3) = 0.
[/tex]
By tracing the restrictions set for the coefficients [itex]a_0,\ldots , a_6[/itex], we arrive at
[tex]
a_1 = r^5 + r^3 + r
[/tex]
I predict some sort of guesswork ahead..
Not enough brainpower left at the moment. To be continued..
 
  • #7
For problem 2
Sorry for bad latex.
The equation can be rearranged to be-
[tex]
f(x)= x^7 + x^5 + x^3 + (c_1^2+c_2^2)x-(c_1^3+c_2^3)=0
[/tex]
By Descartes rule of signs, this problem is trivialized.

[tex]
Let (
c_1^2+c_2^2)=k, k \ge 0
Let
(c_1^3+c_2^3)=t
[\tex]

If t>0, f(x) has exactly one real root, by Descartes rool of signs. If t<0, then f(-x) has exactly one real root, which is the same thing.

https://en.wikipedia.org/wiki/Descartes'_rule_of_signs
 
  • #8
Can I also contribute to maths challenge from next time?
 
  • #9
Question 12 is trivial

N:=213$$21^{39}+39^{21} (mod 9)=0$$

$$21^{39}+39^{21} (mod 5)=1-1=0$$

Hence it is divisible by 45

EDIT: Alternate
21^39 is divisible by 9 because it has 3 raised to a power more than 2. Similarly with 39^21.

Binomial expansion of (20+1)^39= 20^39+ (39)(20^38)+...+(39)(20)+1 which leaves 1 as remainder with five. (40-1)^21 leaves 4 remainder on the other hand. Adding them up leaves no remainder.

Hence, the sum is divisible by 45.

@fresh_42 New proof added. I am in high school BTW (11th grade).
 
Last edited:
  • #10
Pi-is-3 said:
Can I also contribute to maths challenge from next time?
If you like send me a PM, if possible with better LaTeX ;-)
 
  • #11
Pi-is-3 said:
Question 12 is trivial

N:=213$$21^{39}+39^{21} (mod 9)=0$$

$$21^{39}+39^{21} (mod 5)=1-1=0$$

Hence it is divisible by 45
We need a proof, not a computer calculation.
 
  • Haha
Likes Auto-Didact
  • #12
fresh_42 said:
We need a proof, not a computer calculation.
I didn't do a computer calculation. It's simple modular arithmetic. $$a^n (mod k)= (a (mod k))^n$$ for a being integer, and n and k being a natural numbers.
https://brilliant.org/wiki/modular-arithmetic/
 
  • #13
Pi-is-3 said:
I didn't do a computer calculation. It's simple modular arithmetic. $$a^n (mod k)= (a (mod k))^n$$ for a being integer, and n and k being a natural numbers.
https://brilliant.org/wiki/modular-arithmetic/
Can you write this in a form that high schoolers can read and understand it? You do not have to convince me!
 
  • Like
Likes Pi-is-3
  • #14
lriuui0x0 said:
So given ...$$\begin{aligned}
a − b &= \arctan \frac{\tan a − \tan b}{1+ \tan a \tan b} \\
\arctan x - \arctan y &= \arctan \frac{\tan \arctan x - \tan \arctan y}{1 + (\tan \arctan x)(\tan \arctan y)} \\
\arctan x - \arctan y &= \arctan \frac{x - y}{1 + xy}

\end{aligned}$$

Yes, last line is different from what you wrote in post #3 and it is correct now. With this corrected your solution is correct. Well done.
 
  • #15
Regarding Question ##1## there is a simpler solution using just Calculus I. So, anyone other than lriuui0x0 - who has already got the credit, can also take credit if he / she finds it.
 
  • #16
Pi-is-3 said:
For problem 2
Sorry for bad latex.

You're missing an ##x## in your ##f(x)##. Also, please write in Latex - you can take a look at "INFO"##\rightarrow## "Help"##\rightarrow## "LaTeX Primer" (top navigation bar) if you don't know about Latex.

Moreover, the problem states that ##c_1, c_2 \in \mathbb{R}##. What if they are both zero?
 
Last edited:
  • #17
Screen Shot 2019-07-03 at 8.53.36 PM.png

Question 4 :)
 
  • #18
jcloo said:
Question 4 :)

You take the third derivative as I see. What theorem are you using here? How do you get to the last two lines? Also, why is the answer in the form of an image?
 
  • #19
I may have overthought about problem 2.
Fix [itex]u,v\in\mathbb R[/itex] and set
[tex]
f(x) = x^7 + x^5 + x^3 + (1+u^2+v^2)x - (u^3+v^3).
[/tex]
Its derivative is
[tex]
f'(x) = 7x^6 + 5x^4 + 3x^2 + 1+u^2+v^2.
[/tex]
Since [itex]f[/itex] is of odd degree it necessarily has at least one real root by the intermediate value theorem. It also holds that [itex]f'(x) >0[/itex] for every [itex]x\in\mathbb R[/itex]. Thus [itex]f[/itex] is strictly increasing, hence there can only be one real root.
I think my previous idea will fail, because the solution would be given implicitly - thus there being no progress.
Proof by contrapositive. Assume the polynomial has all real roots and set
[tex]
f(x) = (x-a_5)(x-a_4)(x-a_3)(x-a_2)(x-a_1)
[/tex]
We are interested in the coefficients of [itex]x^3, x^4[/itex], we get
[tex]
a = -(a_1+a_2+a_3+a_4+a_5) \\
b = a_1a_2 + a_1a_3 + a_1a_4 + a_1a_5 + a_2a_3 + a_2a_4 + a_2a_5 + a_3a_4 + a_3a_5 + a_4a_5
[/tex]
Study relationship between quantities [itex]2a^2[/itex] and [itex]5b[/itex]. Post expansion we have
[tex]
a^2 = a_1^2 + a_2^2 + a_3^2 + a_4^2 + a_5^2 + \\ 2 \lbrace a_1a_2 + a_1a_3 + a_1a_4 + a_1a_5 + a_2a_3 + a_2a_4 + a_2a_5 + a_3a_4 + a_3a_5 + a_4a_5 \rbrace =: c + 2b.
[/tex]
Finally we have to show [itex]2a^2 \geq 5b[/itex] i.e [itex]2(c+2b) \geq 5b[/itex] i.e [itex]2c \geq b[/itex]. This is somewhat immediate, if a bit technical. We show [itex]4c \geq 2b[/itex]. Group as follows to the left side
[tex]
(a_1^2 - 2a_1a_2 + a_2^2) + (a_1^2 - 2a_1a_3 + a_3^2) + (a_1^2 - 2a_1a_4 + a_4^2) + (a_1^2 - 2a_1a_5 + a_5^2)
[/tex]
with this we have exhausted four copies of [itex]a_1^2[/itex] and the quantity above is non-negative. Proceed with
[tex]
(a_2^2 - 2a_2a_3 + a_3^2) + (a_2^2 - 2a_2a_4 + a_4^2) + (a_2^2 - 2a_2a_5 + a_5^2)
[/tex]
now all copies of [itex]a_2^2[/itex] are exhausted and the quantity remains non-negative. Proceed similarly and obtain [itex]4c - 2b\geq 0[/itex]. This concludes the proof.
I noticed that P4 required the use of calculus? :eek: What, precisely, does that mean? If my approach doesn't qualify, it's fine.
Again, proof by contrapositive. We could make use of the Gauss-Lucas theorem, which states that the zeros of [itex]f'[/itex] are convex combinations of the zeros of [itex]f[/itex]. If we assumed all the roots of [itex]f[/itex] are real, by Gauss Lucas, all the roots of derivatives of arbitrary order are also real. Thus, taking the third derivative
[tex]
f^{(3)}(x) = 60x^2 + 24ax + 6b
[/tex]
and because the roots are real, we have the following restriction:
[tex]
(24a)^2 - 1440b \geq 0 \iff 2a^2 \geq 5b.
[/tex]
 
Last edited:
  • Like
Likes QuantumQuest
  • #20
If I understand this correctly, we need to show that [itex]L[/itex] contains an element [itex]u[/itex] which satisfies
[tex]
\|u\| = \inf _{x\in L} \|x\|
[/tex]
Making use of the parallelogram law we get for any [itex]x,a,b\in H[/itex]
[tex]
\|(x-a) + (x-b)\|^2 + \|(x-a)-(x-b)\|^2 = 2(\|x-a\|^2 + \|x-b\|^2)
[/tex]
Calculating the lhs of the above and by re-arranging we get to
[tex]
\|a-b\|^2 = 2\|x-a\|^2 + 2\|x-b\|^2 - 4\|x- \frac{1}{2}(a+b)\|^2\tag{E}
[/tex]
In light of P10, we can take [itex]x=0[/itex], but we can solve this problem more generally, fix [itex]x\in H[/itex]. Set [itex]l := \inf _{y\in L} \|x-y\| [/itex]. If we take [itex]a,b\in L[/itex] then due to convexity [itex]\frac{1}{2}(a+b) \in L[/itex], therefore
[tex]
l \leq \|x-\frac{1}{2}(a+b)\|.
[/tex]
Per definition of infimum, there exist sequences that converge to it, so pick [itex]a_n\in L, n\in\mathbb N[/itex] such that
[tex]
\|x-a_n\|^2 < l^2 + \frac{1}{n}
[/tex]
By (E) we then have for all [itex]m,n\in \mathbb N[/itex]
[tex]
\|a_n-a_m\| \leq 2\|x-a_n\|^2 + 2\|x-a_m\|^2 -4l^2 \\ \leq 2l^2 + \frac{2}{n} + 2l^2 + \frac{2}{m} - 4l^2 = \frac{2}{n} + \frac{2}{m}
[/tex]
This mean the sequence [itex](a_n)[/itex] is Cauchy and by completeness [itex]a_n \xrightarrow[]{}u \in H[/itex]. Due to closedness [itex]u\in L[/itex]. Consequently
[tex]
\|x-u\|^2 = \lim _{n\to\infty} \|x-a_n\|^2 = l^2.
[/tex]
In fact, this [itex]u[/itex] is unique, for if [itex]v[/itex] satisfies the same demand, then by (E)
[tex]
\|u-v\| \leq 2\|x-u\|^2 + 2\|x-v\|^2 -4l^2 = 2l^2 + 2l^2 - 4l^2 = 0.
[/tex]
Geometrically, this makes sense. For instance, in case of a ball on a plane, for a fixed point in the space, one can always pick a point on the ball closest to that fixed point.
If the fixed point is contained in the ball, the closest point to it is itself, of course. If we give up convexity, the closest point need not be unique. For instance, unit sphere is not convex, there are infinitely many points at distance [itex]1[/itex] from the origin.
 
Last edited:
  • Like
Likes fresh_42
  • #21
QuantumQuest said:
Regarding Question ##1## there is a simpler solution using just Calculus I. So, anyone other than lriuui0x0 - who has already got the credit, can also take credit if he / she finds it.

Let ##f(x) = arctan(x).##

Since ##arctan(x)## is differentiable for all x, by the Mean Value Theorem we know
$$
\frac{f(b) - f(a)}{b - a} = f'(c)
$$
for some c on the interval ##a < c < b##.

##f'(x) = \frac{1}{1 + x^2}##, which is decreasing for ##x > 0##, so for ##0 < a < c < b##, ##f'(b) < f'(c) < f'(a)##.

Substituting, we have
$$
f'(b) < \frac{f(b) - f(a)}{b - a} < f'(a)
$$
$$
\frac{1}{1 + b^2} < \frac{arctan(b) - arctan(a)}{b - a} < \frac{1}{1 + a^2}
$$
$$
\frac{b - a}{1 + b^2} < arctan(b) - arctan(a) < \frac{b - a}{1 + a^2}
$$

Let ##a = 1, b = 4/3##.

$$
\frac{\frac{4}{3} - 1}{1 + (\frac{4}{3})^2} < arctan(4/3) - arctan(1) < \frac{\frac{4}{3} - 1}{1 + 1^2}
$$
$$
\frac{3}{25} < arctan(4/3) - \frac{\pi}{4} < 1/6
$$
$$
\frac{\pi}{4} + \frac{3}{25} < arctan(4/3) < \frac{\pi}{4} + \frac{1}{6}
$$
 
  • Like
Likes mfb, QuantumQuest and fresh_42
  • #22
My solution to 11.
246120


By the way i wanted to know whether can we solve the same integral but indefinite.If we can please provide a solution.
 
  • #23
Physics lover said:
My solution to 11.
View attachment 246120

By the way i wanted to know whether can we solve the same integral but indefinite.If we can please provide a solution.

Why is there +c? It is a definite integral! For the rest,you are correct. I doubt you can find an elementary primitive. This was the solution I had in mind.
 
  • Like
Likes fresh_42
  • #24
Math_QED said:
Why is there +c? It is a definite integral! For the rest,you are correct. I doubt you can find an elementary primitive. This was the solution I had in mind.
Oh sorry for that c.Ok i will try for another method Can we solve its indefinite one?
 
  • #26
nuuskur said:
I may have overthought about problem 2.

Yes indeed. The solution you give in post #19 using simple calculus is much faster and simpler. Well done.

nuuskur said:
I noticed that P4 required the use of calculus? :eek: What, precisely, does that mean? If my approach doesn't qualify, it's fine.

There are various ways that can be used but I ask for the use of calculus as it is a fast and simple approach. Your first solution for question ##4## gets unnecessarily complex but the second is fine.
 
  • #27
Flatlanderr said:
Let ##f(x) = arctan(x).##

Since ##arctan(x)## is differentiable for all x, by the Mean Value Theorem we know
$$
\frac{f(b) - f(a)}{b - a} = f'(c)
$$
for some c on the interval ##a < c < b##.

##f'(x) = \frac{1}{1 + x^2}##, which is decreasing for ##x > 0##, so for ##0 < a < c < b##, ##f'(b) < f'(c) < f'(a)##.

Substituting, we have
$$
f'(b) < \frac{f(b) - f(a)}{b - a} < f'(a)
$$
$$
\frac{1}{1 + b^2} < \frac{arctan(b) - arctan(a)}{b - a} < \frac{1}{1 + a^2}
$$
$$
\frac{b - a}{1 + b^2} < arctan(b) - arctan(a) < \frac{b - a}{1 + a^2}
$$

Let ##a = 1, b = 4/3##.

$$
\frac{\frac{4}{3} - 1}{1 + (\frac{4}{3})^2} < arctan(4/3) - arctan(1) < \frac{\frac{4}{3} - 1}{1 + 1^2}
$$
$$
\frac{3}{25} < arctan(4/3) - \frac{\pi}{4} < 1/6
$$
$$
\frac{\pi}{4} + \frac{3}{25} < arctan(4/3) < \frac{\pi}{4} + \frac{1}{6}
$$

Well done @Flatlanderr
 
  • #28
Physics lover said:
My solution to 11.
View attachment 246120

By the way i wanted to know whether can we solve the same integral but indefinite.If we can please provide a solution.
How you change ##x \to \frac{\pi}{2}-x## without changing interval of integration?
 
  • Like
Likes Pi-is-3
  • #29
LagrangeEuler said:
How you change ##x \to \frac{\pi}{2}-x## without changing interval of integration?

The new integration bounds are from ##\pi/2## to ##0## and we get a minus sign for the integral as a result of the substitution. Switching the integral bounds removes the minus sign.
 
  • Informative
Likes Pi-is-3
  • #30
LagrangeEuler said:
How you change ##x \to \frac{\pi}{2}-x## without changing interval of integration?
This is a propertie of definite integral
##\int_0^a f(x) \, dx =## ##\int_0^a f(0+a-x) \, dx ##
 
  • Like
Likes Pi-is-3
  • #31
  • #32
I am having a lot of problem with latex. I am not that bad at latex in AoPS or Stack exchange but I am not able to use it properly here :(. Example, my first line of answer should be this-

By AM-GM

$$ \frac{(u+(1-v)+v+(1-w)+w+(1-u))}{6} \geq
(u(1-v)v(1-w)w(1-u))^{\frac{1}{6}$$

but the Latex is not working. What am I doing wrong?
 
  • #33
Pi-is-3 said:
I am having a lot of problem with latex. I am not that bad at latex in AoPS or Stack exchange but I am not able to use it properly here :(. Example, my first line of answer should be this-

By AM-GM

$$ \frac{(u+(1-v)+v+(1-w)+w+(1-u))}{6} \geq
(u(1-v)v(1-w)w(1-u))^{\frac{1}{6}$$

but the Latex is not working. What am I doing wrong?
This is due to your laziness. If you copy and paste instead of actually writing it, you also copy hidden control sequences as colors or fonts. This doesn't work in LaTeX here.

If it helps you, you can download a script program, e.g. AutoHotkey, which allows you to abbreviate certain key sequences by keyboard shortcuts. For example I have \frac{}{} on Ctrl+F or \ begin{bmatrix} \ end{bmatrix} on Alt+M (without the blanks).
 
  • #34
@fresh_42 Thanks for telling my mistake! Now I can latex again 😁
By AM-GM

$$ \frac {(u)+(1-v)+(v)+(1-w)+(w)+(1-u)} {6} \geq [(u)(1-v)(v)(1-w)(w)(1-u)]^{\frac{1}{6}}$$

Implies

$$ \frac {1} {2} \geq [(u)(1-v)(v)(1-w)(w)(1-u)]^{\frac{1}{6}}$$

Squaring both sides (works in this inequality as both sides are positive)

$$ \frac {1} {4} \geq [(u)(1-v)(v)(1-w)(w)(1-u)]^{\frac{1}{3}}$$

Now using GM-HM

$$ \frac {1} {4} \geq [(u)(1-v)(v)(1-w)(w)(1-u)]^{\frac{1}{3}} \geq \frac {3} {\frac{1}{u(1-v)}+\frac{1}{v(1-w)} + \frac{1}{w(1-u)}}$$

Implies

$$ \frac{1}{u(1-v)} + \frac{1}{v(1-w)} + \frac{1}{w(1-u)} \geq 12 $$

W.L.O.G assume ## u(1-v) \leq v(1-w) \leq w(1-u) ##

then ## \frac{3}{u(1-v)} \geq 12 ##

implies $$\frac{1}{4} \geq u(1-v) $$

Hence Proved
 
  • #35
Pi-is-3 said:
@fresh_42 Thanks for telling my mistake! Now I can latex again 😁
By AM-GM

$$ \frac {(u)+(1-v)+(v)+(1-w)+(w)+(1-u)} {6} \geq [(u)(1-v)(v)(1-w)(w)(1-u)]^{\frac{1}{6}}$$

Implies

$$ \frac {1} {2} \geq [(u)(1-v)(v)(1-w)(w)(1-u)]^{\frac{1}{6}}$$

Squaring both sides (works in this inequality as both sides are positive)

$$ \frac {1} {4} \geq [(u)(1-v)(v)(1-w)(w)(1-u)]^{\frac{1}{3}}$$

Now using GM-HM

$$ \frac {1} {4} \geq [(u)(1-v)(v)(1-w)(w)(1-u)]^{\frac{1}{3}} \geq \frac {3} {\frac{1}{u(1-v)}+\frac{1}{v(1-w)} + \frac{1}{w(1-u)}}$$

Implies

$$ \frac{1}{u(1-v)} + \frac{1}{v(1-w)} + \frac{1}{w(1-u)} \geq 12 $$

W.L.O.G assume ## u(1-v) \leq v(1-w) \leq w(1-u) ##

then ## \frac{3}{u(1-v)} \geq 12 ##

implies $$\frac{1}{4} \geq u(1-v) $$

Hence Proved
Well done and far better written than your first attempts, and I do not mean LaTeX, I mean the structure of your proof!

You should consider to download this little helper. It saves so much time, that I can almost type at a normal speed without all these special characters. I even use keys for \alpha , \beta, \omega etc.

If anyone still wants to try: there is another proof using a binomial formula.
 
Last edited:
  • Like
Likes Pi-is-3

Similar threads

  • Math Proof Training and Practice
2
Replies
42
Views
6K
  • Math Proof Training and Practice
3
Replies
80
Views
4K
  • Math Proof Training and Practice
2
Replies
69
Views
3K
  • Math Proof Training and Practice
4
Replies
121
Views
18K
  • Math Proof Training and Practice
3
Replies
93
Views
10K
  • Math Proof Training and Practice
4
Replies
137
Views
15K
  • Math Proof Training and Practice
3
Replies
104
Views
13K
  • Math Proof Training and Practice
3
Replies
100
Views
7K
  • Math Proof Training and Practice
2
Replies
67
Views
7K
  • Math Proof Training and Practice
4
Replies
114
Views
6K
Back
Top